¿Quiéres el ejercicio resuelto en menos de 48 horas? Paga desde $US 4 (4 DÓLARES)  vía   PayPal   o   desde  $ 10.000 pesos (colombianos) vía Nequi si estás en Colombia, comunicándote al whatsapp +573203806207 para confirmar pago, y tendrás el ejercicio resuelto.

SOLUCIÓN DE EJERCICIOS

FÍSICA PARA CIENCIAS E INGENIERÍA  – SERWAY

SOLUCIÓN PROBLEMA 16 CAPÍTULO 25 FÍSICA SERWAY

16.  Dos partículas con carga, cada una de ellas con una magnitud de 2.0 uC, se localizan en el eje de las x. Una está a x =1.00 m, y la otra está a x= -1.00 m. a) Determine el potencial eléctrico sobre el eje de las y en el punto y= 0.500 m. b) Calcule el cambio en la energía potencial eléctrica del sistema al traer una tercera carga de -3.00 uC desde un punto infinitamente lejano a una posición en el eje de las y en y= 0.500 m.

Solución:

Solución 1: blog Mistersolución

CONSULTA AQUÍ OTROS EJERCICIOS RESUELTOS DEL LIBRO FÍSICA SERWAY VOLUMEN 1.

¿Te sirvió el ejercicio? Compártelo

¿Tienes Dudas o cosas que agregar? Comenta

¿El ejercicio aún no está resuelto? Solicítalo comentando aquí y nuestra comunidad lo resolverá rápidamente. Si tienes la solución ¡Envíala! La comunidad estará agradecida.

¿Quiéres el ejercicio resuelto en menos de 48 horas? Paga desde $US 4 (4 DÓLARES)  vía   PayPal   o   desde  $ 10.000 pesos (colombianos) vía Nequi si estás en Colombia, comunicándote al whatsapp +573203806207 para confirmar pago, y tendrás el ejercicio resuelto.

Deja una respuesta